Integration. (1 Viewer)

Drongoski

Well-Known Member
Joined
Feb 22, 2009
Messages
4,249
Gender
Male
HSC
N/A
Your answer is right but your upper limit is meant to be 1/2.ln(3)
Yes but 0.5 ln 3 is the same as ln(sqrt(3)); otherwise it'd be very unlikely I'd have got the right answer.
 

Users Who Are Viewing This Thread (Users: 0, Guests: 1)

Top